How do I solve this $z^5 + z^3 = iz^2 + i$












-3












$begingroup$


How can I find out what z is and the zero of this function? $z^5 + z^3 = iz^2 + i$



I tried everything but I cannot solve it...



Thanks in advance!










share|cite|improve this question











$endgroup$












  • $begingroup$
    Is my suggested edit correct?
    $endgroup$
    – AryanSonwatikar
    Dec 9 '18 at 15:24






  • 4




    $begingroup$
    Hint $z^3(z^2+1)=i(z^2+1)$
    $endgroup$
    – egreg
    Dec 9 '18 at 15:26
















-3












$begingroup$


How can I find out what z is and the zero of this function? $z^5 + z^3 = iz^2 + i$



I tried everything but I cannot solve it...



Thanks in advance!










share|cite|improve this question











$endgroup$












  • $begingroup$
    Is my suggested edit correct?
    $endgroup$
    – AryanSonwatikar
    Dec 9 '18 at 15:24






  • 4




    $begingroup$
    Hint $z^3(z^2+1)=i(z^2+1)$
    $endgroup$
    – egreg
    Dec 9 '18 at 15:26














-3












-3








-3





$begingroup$


How can I find out what z is and the zero of this function? $z^5 + z^3 = iz^2 + i$



I tried everything but I cannot solve it...



Thanks in advance!










share|cite|improve this question











$endgroup$




How can I find out what z is and the zero of this function? $z^5 + z^3 = iz^2 + i$



I tried everything but I cannot solve it...



Thanks in advance!







complex-numbers






share|cite|improve this question















share|cite|improve this question













share|cite|improve this question




share|cite|improve this question








edited Dec 9 '18 at 15:26







jamyolive

















asked Dec 9 '18 at 15:19









jamyolivejamyolive

246




246












  • $begingroup$
    Is my suggested edit correct?
    $endgroup$
    – AryanSonwatikar
    Dec 9 '18 at 15:24






  • 4




    $begingroup$
    Hint $z^3(z^2+1)=i(z^2+1)$
    $endgroup$
    – egreg
    Dec 9 '18 at 15:26


















  • $begingroup$
    Is my suggested edit correct?
    $endgroup$
    – AryanSonwatikar
    Dec 9 '18 at 15:24






  • 4




    $begingroup$
    Hint $z^3(z^2+1)=i(z^2+1)$
    $endgroup$
    – egreg
    Dec 9 '18 at 15:26
















$begingroup$
Is my suggested edit correct?
$endgroup$
– AryanSonwatikar
Dec 9 '18 at 15:24




$begingroup$
Is my suggested edit correct?
$endgroup$
– AryanSonwatikar
Dec 9 '18 at 15:24




4




4




$begingroup$
Hint $z^3(z^2+1)=i(z^2+1)$
$endgroup$
– egreg
Dec 9 '18 at 15:26




$begingroup$
Hint $z^3(z^2+1)=i(z^2+1)$
$endgroup$
– egreg
Dec 9 '18 at 15:26










1 Answer
1






active

oldest

votes


















0












$begingroup$

HINT:
$$z^5+z^3-iz^2-i=(z^2+1)(z^3-i)=0$$






share|cite|improve this answer









$endgroup$













    Your Answer





    StackExchange.ifUsing("editor", function () {
    return StackExchange.using("mathjaxEditing", function () {
    StackExchange.MarkdownEditor.creationCallbacks.add(function (editor, postfix) {
    StackExchange.mathjaxEditing.prepareWmdForMathJax(editor, postfix, [["$", "$"], ["\\(","\\)"]]);
    });
    });
    }, "mathjax-editing");

    StackExchange.ready(function() {
    var channelOptions = {
    tags: "".split(" "),
    id: "69"
    };
    initTagRenderer("".split(" "), "".split(" "), channelOptions);

    StackExchange.using("externalEditor", function() {
    // Have to fire editor after snippets, if snippets enabled
    if (StackExchange.settings.snippets.snippetsEnabled) {
    StackExchange.using("snippets", function() {
    createEditor();
    });
    }
    else {
    createEditor();
    }
    });

    function createEditor() {
    StackExchange.prepareEditor({
    heartbeatType: 'answer',
    autoActivateHeartbeat: false,
    convertImagesToLinks: true,
    noModals: true,
    showLowRepImageUploadWarning: true,
    reputationToPostImages: 10,
    bindNavPrevention: true,
    postfix: "",
    imageUploader: {
    brandingHtml: "Powered by u003ca class="icon-imgur-white" href="https://imgur.com/"u003eu003c/au003e",
    contentPolicyHtml: "User contributions licensed under u003ca href="https://creativecommons.org/licenses/by-sa/3.0/"u003ecc by-sa 3.0 with attribution requiredu003c/au003e u003ca href="https://stackoverflow.com/legal/content-policy"u003e(content policy)u003c/au003e",
    allowUrls: true
    },
    noCode: true, onDemand: true,
    discardSelector: ".discard-answer"
    ,immediatelyShowMarkdownHelp:true
    });


    }
    });














    draft saved

    draft discarded


















    StackExchange.ready(
    function () {
    StackExchange.openid.initPostLogin('.new-post-login', 'https%3a%2f%2fmath.stackexchange.com%2fquestions%2f3032491%2fhow-do-i-solve-this-z5-z3-iz2-i%23new-answer', 'question_page');
    }
    );

    Post as a guest















    Required, but never shown

























    1 Answer
    1






    active

    oldest

    votes








    1 Answer
    1






    active

    oldest

    votes









    active

    oldest

    votes






    active

    oldest

    votes









    0












    $begingroup$

    HINT:
    $$z^5+z^3-iz^2-i=(z^2+1)(z^3-i)=0$$






    share|cite|improve this answer









    $endgroup$


















      0












      $begingroup$

      HINT:
      $$z^5+z^3-iz^2-i=(z^2+1)(z^3-i)=0$$






      share|cite|improve this answer









      $endgroup$
















        0












        0








        0





        $begingroup$

        HINT:
        $$z^5+z^3-iz^2-i=(z^2+1)(z^3-i)=0$$






        share|cite|improve this answer









        $endgroup$



        HINT:
        $$z^5+z^3-iz^2-i=(z^2+1)(z^3-i)=0$$







        share|cite|improve this answer












        share|cite|improve this answer



        share|cite|improve this answer










        answered Dec 9 '18 at 15:42









        JJacquelinJJacquelin

        45k21855




        45k21855






























            draft saved

            draft discarded




















































            Thanks for contributing an answer to Mathematics Stack Exchange!


            • Please be sure to answer the question. Provide details and share your research!

            But avoid



            • Asking for help, clarification, or responding to other answers.

            • Making statements based on opinion; back them up with references or personal experience.


            Use MathJax to format equations. MathJax reference.


            To learn more, see our tips on writing great answers.




            draft saved


            draft discarded














            StackExchange.ready(
            function () {
            StackExchange.openid.initPostLogin('.new-post-login', 'https%3a%2f%2fmath.stackexchange.com%2fquestions%2f3032491%2fhow-do-i-solve-this-z5-z3-iz2-i%23new-answer', 'question_page');
            }
            );

            Post as a guest















            Required, but never shown





















































            Required, but never shown














            Required, but never shown












            Required, but never shown







            Required, but never shown

































            Required, but never shown














            Required, but never shown












            Required, but never shown







            Required, but never shown







            Popular posts from this blog

            Biblatex bibliography style without URLs when DOI exists (in Overleaf with Zotero bibliography)

            ComboBox Display Member on multiple fields

            Is it possible to collect Nectar points via Trainline?